Answer (D) is correct . According to the EMH, stock prices are in equilibrium and investors cannot obtain abnormal returns, that is, returns in excess of the riskiness of their investments. The semistrong form of the EMH postulates that current market prices reflect all publicly available information. However, investors with inside information can still earn an abnormal return.
Answer (A) is incorrect because The EMH states that current prices reflect at least the information contained in past price movements. Answer (B) is incorrect because The strong form of the EMH states that current market prices reflect all pertinent information, including insider information. Answer (C) is incorrect because The weak form of the EMH states that current market prices reflect only information contained in past price movements.
|